Put the following equation of a line into slope-intercept form, simplifying all
fractions.
18x + 3y = -18

Answers

Answer 1

Answer:

y= -6x-6, I think, hope it helped

Step-by-step explanation:


Related Questions

Find the equation of the linear function represented by the table below in slope-intercept form.

Answers

Answer:

y=-4x-5

Step-by-step explanation:

The slope of the line is - 4, the equation of line is y=-4x-5

John runs a computer software store. Yesterday he counted 140 people who walked by the store, 63 of whom came into the store. Of the 63, only 25 bought something in the store.
(a) Estimate the probability that a person who walks by the store will enter the store. (Round your answer to two decimal places.)
(b) Estimate the probability that a person who walks into the store will buy something. (Round your answer to two decimal places.)
(c) Estimate the probability that a person who walks by the store will come in and buy something. (Round your answer to two decimal places.)
(d) Estimate the probability that a person who comes into the store will buy nothing. (Round your answer to two decimal places.)

Answers

Answer:

.................

Step-by-step explanation:

............

a. 45%
b. 39.68%
c. 17.86%
d. 60.32%

look at the image for the question

Answers

It’s 16
You would multiply height (4) by the base (length(2) x width(2)=4) which is 16

1. A helicopter is at a position from two VORS (VHF Omnidirectional
Radio Range, an aircraft navigation system operating in the VHF band -
not covered in chapter) as in the diagram shown below. Given the angles
shown, find the third angle.
Helicopter
74.0°
66.0°
VOR
VOR

Answers

The position of the helicopter and the two VORs forms a triangle and the third angle formed by these three entities is 40 degrees

The diagram is not shown; however, the question can still be answered.

The given angles are:

[tex]\theta_1 = 74.0^o[/tex]

[tex]\theta_2 = 66.0^o[/tex]

Represent the third angle as [tex]\theta_3[/tex]

The helicopter and the 2 VORs form a triangle.

So, we make use of the following theorem to calculate the third angle

[tex]\theta_1 + \theta_2 + \theta_3= 180^o[/tex] ---- sum of angles in a triangle

Substitute known values

[tex]74.0^o + 66.0^o + \theta_3= 180^o[/tex]

[tex]140.0^o + \theta_3= 180^o[/tex]

Collect like terms

[tex]\theta_3= 180 -140.0^o[/tex]

[tex]\theta_3= 40^o[/tex]

Hence, the third angle is 40 degrees.

Learn more about angles in a triangle at:

https://brainly.com/question/14780489

Find the area of the figure. (Sides meet at right angles.)

Answers

Answer:

56

Step-by-step explanation:

A=(3*4)+(4*(4+3+4))=56

Find an upper bound for E(h) the error of the machine approximation of the two-point forward difference formula for the first derivative and then find the h corresponding to the minimum of E(h).

The two-point forward difference formula for f'(x) is:_________

Answers

Answer:

I doubt it is not going to be a great

Please Help!! Whoever helps and gets it correct gets Brainliest and 5 star rating!!

Answers

Answer:

the reasoning states that "all the numbers begin with a 7 or an 8"

however this is not accurate as they can be in different placements

which can make a big difference in the total estimate.

for example:

the number could've been an 8, or an 80

they both begin with an 8

however have totally different values and could have messed up the total estimated number.

hope this helps :D

Seven and one-half foot-pounds of work is required to compress a spring 2 inches from its natural length. Find the work required to compress the spring an additional 3 inch.

Answers

Answer:

Apply Hooke's Law to the integral application for work: W = int_a^b F dx , we get:

W = int_a^b kx dx

W = k * int_a^b x dx

Apply Power rule for integration: int x^n(dx) = x^(n+1)/(n+1)

W = k * x^(1+1)/(1+1)|_a^b

W = k * x^2/2|_a^b

 

From the given work: seven and one-half foot-pounds (7.5 ft-lbs) , note that the units has "ft" instead of inches.   To be consistent, apply the conversion factor: 12 inches = 1 foot then:

 

2 inches = 1/6 ft

 

1/2 or 0.5 inches =1/24 ft

To solve for k, we consider the initial condition of applying 7.5 ft-lbs to compress a spring  2 inches or 1/6 ft from its natural length. Compressing 1/6 ft of it natural length implies the boundary values: a=0 to b=1/6 ft.

Applying  W = k * x^2/2|_a^b , we get:

7.5= k * x^2/2|_0^(1/6)

Apply definite integral formula: F(x)|_a^b = F(b)-F(a) .

7.5 =k [(1/6)^2/2-(0)^2/2]

7.5 = k * [(1/36)/2 -0]

7.5= k *[1/72]

 

k =7.5*72

k =540

 

To solve for the work needed to compress the spring with additional 1/24 ft, we  plug-in: k =540 , a=1/6 , and b = 5/24 on W = k * x^2/2|_a^b .

Note that compressing "additional one-half inches" from its 2 inches compression is the same as to  compress a spring 2.5 inches or 5/24 ft from its natural length.

W= 540 * x^2/2|_((1/6))^((5/24))

W = 540 [ (5/24)^2/2-(1/6)^2/2 ]

W =540 [25/1152- 1/72 ]

W =540[1/128]

W=135/32 or 4.21875 ft-lbs

Step-by-step explanation:

Round 573.073 to the greatest place

Answers

Answer:

574

Step-by-step explanation:

To round a two-digit number to the nearest ten, simply increase it or decrease it to the nearest number that ends in 0: When a number ends in 1, 2, 3, or 4, bring it down; in other words, keep the tens digit the same and turn the ones digit into a 0

Hope this helps <3

if U>T, R>Q, S>T and T>R, which of the following is TRUE?
1. S>Q
2. U > S
3.U > R​
A. 1 only
B. 2 only
C. 1 and 2
D. 2 and 3

Answers

Answer:

C. 1 and 2

Step-by-step explantation:

First, i would order them as U>T, T>R, R>Q, S>T

we can rewrite them as

U>T>R>Q,

now adding S, we get U>S>T>R>Q,

so U>S

We can also rewrite all of them as inequalities:

U-T>0

T-R>0

R-Q>0

S-T>0

Add R-Q and T-R

(R-Q)+(T-R)>0

-Q+T>0

T>Q, but because S>T we can say S>Q

Find 0.2B
B=[50 10
25 15]

Answers

Multiplying a matrix by a scalar results in every entry in a matrix get multiplied by that scalar, as defined,

[tex]a\begin{bmatrix}b&c\\d&e\\\end{bmatrix}=\begin{bmatrix}ab&ac\\ad&ae\\\end{bmatrix}[/tex]

So in our case, ([tex]0.2=\frac{1}{5}[/tex]

[tex]\frac{1}{5}\begin{bmatrix}50&10\\25&15\\\end{bmatrix}=\begin{bmatrix}\frac{50}{5}&\frac{10}{5}\\\frac{25}{5}&\frac{15}{5}\\\end{bmatrix}=\boxed{\begin{bmatrix}10&2\\5&3\\\end{bmatrix}}[/tex]

Hope this helps :)

Paul can install a 300-square-foot hardwood floor in 18 hours. Matt can install the same floor in 22 hours. How long would it take Paul and Matt to install the floor working together?
4 hours
9.9 hours
13.2 hours
30 hours

Answers

Answer:

9.9 hours

Step-by-step explanation:

The formula to determine the time together is

1/a+1/b = 1/c  where a and b are the times alone and c is the time together

1/18 + 1/22 = 1/c

The least common multiply of the denominators is 198c

198c(1/18 + 1/22 = 1/c)

11c+ 9c = 198

20c = 198

Divide by 20

20c/20 =198/20

c =9.9

Answer:

B - 9.9 hrs

Step-by-step explanation:

took the test.

Solve for X and show your work and explain please

Answers

Answer: x = 45

Step-by-step explanation:

Given

(2/3)x + 4 = (4/5)x - 2

Add 2 on both sides

(2/3)x + 4 + 2 = (4/5)x - 2 + 2

(2/3)x + 6 = (4/5)x

Subtract (2/3)x on both sides

(2/3)x + 6 - (2/3)x = (4/5)x - (2/3)x

6 = (12/15)x - (10/15)x

6 = (2/15)x

Divide 2/15 on both sides

6 / (2/15) = (2/15)x / (2/15)

[tex]\boxed{x=45}[/tex]

Hope this helps!! :)

Please let me know if you have any questions

Answer:

x = 45

Step-by-step explanation:

2/3 x + 4 = 4/5x - 2             Add 2 to both sides

2/3 x + 4 + 2 = 4/5x            Combine

2/3x + 6 = 4/5x                   Subtract 2/3 x from both sides.

6 = 4/5x - 2/3 x                  Multiply both sides by 15

6*15 = 4/5 x * 15 - 2/3x * 15

6*15 = 12x - 10x                   Combine the left and right

90 = 2x                               Divide by 2

x = 45

Let's see if it works.

LHS = 2/3 * 45 + 4

LHS = 2*15 + 4

LHS = 30 + 4

LHS = 34

RHS

Right hand side = 4/5 * 45 - 2

RHS = 36 - 2

RHS = 34 which is the same as the LHS

8.9 x 10^3 in standard notation

Answers

Answer:

that is n standard notation mah frand

8.9 × 10^3 being scientific notation of " 8900 "

[tex]\huge\text{Hey there!}[/tex]

[tex]\large\textsf{8.9}\times\large\textsf{10}^\mathsf{3}\\\\\mathsf{10^3}\\\mathsf{= 10\times10\times10}\\\mathsf{= 100\times10}\\\mathsf{= \bf 1,000}\\\\\large\textsf{8.9}\times\large\textsf{1,000}\\\\\large\textsf{= \bf 8,900}\\\\\\\boxed{\boxed{\huge\text{Answer: \boxed{\underline{\underline{\bf 8,900}}}}}}\huge\checkmark[/tex]

[tex]\huge\text{Good luck on your assignment \& enjoy your day!}[/tex]

~[tex]\boxed{\huge\text{}\boxed{\frak{Amphitrite1040:)}}}[/tex]

find the missing side of the triangle

Answers

Answer:

x = 34

Step-by-step explanation:

Pytago:

x[tex]30^{2} + 16^{2} = x^2\\x = \sqrt{30^2 + 16^2} \\x = 34[/tex]

Find an equation of the plane orthogonal to the line
(x,y,z)=(0,9,6)+t(7,−7,−6)

which passes through the point (9, 6, 0).

Give your answer in the form ax+by+cz=d (with a=7).

Answers

The given line is orthogonal to the plane you want to find, so the tangent vector of this line can be used as the normal vector for the plane.

The tangent vector for the line is

d/dt (⟨0, 9, 6⟩ + ⟨7, -7, -6⟩t ) = ⟨7, -7, -6⟩

Then the plane that passes through the origin with this as its normal vector has equation

x, y, z⟩ • ⟨7, -7, -6⟩ = 0

We want the plane to pass through the point (9, 6, 0), so we just translate every vector pointing to the plane itself by adding ⟨9, 6, 0⟩,

(⟨x, y, z⟩ - ⟨9, 6, 0⟩) • ⟨7, -7, -6⟩ = 0

Simplifying this expression and writing it standard form gives

x - 9, y - 6, z⟩ • ⟨7, -7, -6⟩ = 0

7 (x - 9) - 7 (y - 6) - 6z = 0

7x - 63 - 7y + 42 - 6z = 0

7x - 7y - 6z = 21

so that

a = 7, b = -7, c = -6, and d = 21

An equation of the plane orthogonal to the line 7x - 7y - 6z = 21.

The given line is orthogonal to the plane you want to find,

So the tangent vector of this line can be used as

The normal vector for the plane.

The tangent vector for the line is,

What is the tangent vector?

A tangent vector is a vector that is tangent to a curve or surface at a given point.

d/dt (⟨0, 9, 6⟩ + ⟨7, -7, -6⟩t ) = ⟨7, -7, -6⟩

Then the plane that passes through the origin with this as its normal vector has the equation

⟨x, y, z⟩ • ⟨7, -7, -6⟩ = 0

We want the plane to pass through the point (9, 6, 0), so we just

translate every vector pointing to the plane itself by adding ⟨9, 6, 0⟩,

(⟨x, y, z⟩ - ⟨9, 6, 0⟩) • ⟨7, -7, -6⟩ = 0

Simplifying this expression and writing it in standard form gives

⟨x - 9, y - 6, z⟩ • ⟨7, -7, -6⟩ = 0

7 (x - 9) - 7 (y - 6) - 6z = 0

7x - 63 - 7y + 42 - 6z = 0

7x - 7y - 6z = 21

So that, a = 7, b = -7, c = -6, and d = 21.

To learn more about the equation of plane visit:

https://brainly.com/question/1603217

State if the scenario involves a permutation or a combination. Then find the number of possibilities.

A team of 15 basketball players needs to choose two players to refill the water cooler.

Permutation/Combination:

Answer:

Answers

Answer:

Permutation ; 210 ways

Step-by-step explanation:

Permutation and combination methods refers to mathematical solution to finding the number of ways of making selection for a group of objects.

Usually, selection process whereby the order of selection does not matter are being treated using permutation, while those which takes the order of selection into cognizance are calculated using combination.

Here, selecting 2 players from 15 ; since order does not matter, we use permutation ;

Recall :

nPr = n! ÷ (n - r)!

Hence,

15P2 = 15! ÷ (15 - 2)!

15P2 = 15! ÷ 13!

15P2 = (15 * 14) = 210 ways

which of the following illustrates commutative property of addition? 17+4=4+17​

Answers

9514 1404 393

Answer:

  17 +4 = 4 +17

Step-by-step explanation:

The only expression shown here illustrates that property.

Reason Can you subtract a positive integer from a positive integer
and get a negive result? Explain your answer.

Answers

Answer:

No

Step-by-step explanation:

No matter the situation, when you multiply a negative by a negativeyou get a positive and a positive by a positive you get a positive. but if its two different like a negative and a positive then its NEGITIVE.

let's say you have 23 and you're multiplying by 2.

It's always increasing so it doesnt ever reach the negitive numbers.

Find the length of the arc.

A. 539π/12 km
B. 9π/3 km
C. 9π/2 km
D. 18π km

Answers

Answer:

b because it is I found out cus I took test

The length of the arc 9π/2 km.

The answer is option C.9π/2 km.

What is the arc of the circle?

The arc period of a circle can be calculated with the radius and relevant perspective using the arc period method.

  ⇒angle= arc/radius

     ⇒  135°=arc/6km

     ⇒ arc =135°*6km

     ⇒arc=135°*π/180° * 6km

    ⇒arc = 9π/2 km

Learn more about circle here:-https://brainly.com/question/24375372

#SPJ2

Find the slope of the line that goes through the
(2,6) and (-1, -6)

Answers

We can use the formula y2-y1/x2-x1 to get our slope. y2 and x2 are our second y and x coordinates, meanwhile y1 and x1 are our first y and x coordinates. -6-6/-1 -2 is -12/-3. -12/-3 is 4, the slope is 4.
the slope is 1/4 because you use the slope intercept formula

please solve the question ​

Answers

Answer:

[tex]g(-1) = -1[/tex]

[tex]g(0.75) = 0[/tex]

[tex]g(1)= 1[/tex]

Step-by-step explanation:

Given

See attachment

Solving (a): g(-1)

We make use of:

[tex]g(x) = -1[/tex]

Because: [tex]-1 \le x < 0[/tex] is true for x =-1

Hence:

[tex]g(-1) = -1[/tex]

Solving (b): g(0.75)

We make use of:

[tex]g(x) = 0[/tex]

Because: [tex]0 \le x < 1[/tex] is true for x =0.75

Hence:

[tex]g(0.75) = 0[/tex]

Solving (b): g(1)

We make use of:

[tex]g(x) = 1[/tex]

Because: [tex]1 \le x < 2[/tex] is true for x =1

Hence:

[tex]g(1)= 1[/tex]

Solve for x: 10/3 = x/(−5/2)

Answers

9514 1404 393

Answer:

  x = -25/3

Step-by-step explanation:

Multiply by the inverse of the coefficient of x. Reduce the fraction.

  (-5/2)(10/3) = (-5/2)(x/(-5/2))

  -50/6 = x = -25/3

Answer:

-25/3

Step-by-step explanation:

the other person is also correct. khan said so

Which ratio is equal to 27 : 81?

Answers

3:9 and if you reduce it again, 1:3

Answer:

1:3

Step-by-step explanation:

27 : 81

Divide each side by 27

27/27 : 81/27

1:3

Identify the transformed function that represents f(x) = ln x stretched vertically by a factor of 17, reflected across the x-axis, and shifted by 19 units left.
A. g(x) = −17ln (x + 19)
B. g(x) = 17ln (x − 19)
C. g(x) = 17ln (x + 19)
D. g(x) = −17ln (x − 19)

Answers

Answer:

b

Step-by-step explanation:

ANSWER. EXPLANATION. The given logarithmic function is. The transformation,. stretches the graph of y=f(x) vertically by a factor of c units ...

4 votes

ANSWER[tex]y = - 3 ln(x - 7) [/tex]EXPLANATIONThe given logarithmic function is [tex]f(x) = ln(x) [/tex]The transformation, [tex]y = - cf(x - k)[/tex]stretches

The length of a rectangle is twice its width. If the area of the rectangle is 72in², find its perimeter

Answers

Let breadth be x

Length=2x

[tex]\\ \sf\longmapsto Area=Length\times Breadth[/tex]

[tex]\\ \sf\longmapsto 72=2x(x)[/tex]

[tex]\\ \sf\longmapsto 2x^2=72[/tex]

[tex]\\ \sf\longmapsto x^2=\dfrac{72}{2}[/tex]

[tex]\\ \sf\longmapsto x^2=36[/tex]

[tex]\\ \sf\longmapsto x=\sqrt{36}[/tex]

[tex]\\ \sf\longmapsto x=6[/tex]

Length=6×2=12inBreadth=6in

[tex]\\ \sf\longmapsto Perimeter=2(L+B)[/tex]

[tex]\\ \sf\longmapsto Perimeter=2(12+6)[/tex]

[tex]\\ \sf\longmapsto Perimeter=2(18)[/tex]

[tex]\\ \sf\longmapsto Perimeter=36in[/tex]

Graph y=|x|+5, how does it compare to parent graph y=|x|

Answers

9514 1404 393

Answer:

  it is shifted 5 units upward

Step-by-step explanation:

The y-coordinate is a measure of the distance above the x-axis. When 5 is added to a y-coordinate, the point is shifted 5 units upward.

The function y = |x| +5 adds 5 units to the y-value of every point of the graph of y = |x|. The graph of y=|x|+5 is shifted 5 units upward from the parent graph.

In a class of 70 pupils, 36 like tasty time , 34 like ice-
cream, 6 like both tasty time }
draw a Venn diagram to show the data.
find how
many
like neither tasty time nor ice-cream

Answers

Step-by-step explanation:

I think this might be the correct answer

The number of pupils that like neither tasty-time nor ice cream is 6 if in a class of 70 pupils, 36 like tasty time, 34 like ice cream, 6 like both tasty times.

What is the Venn diagram?

It is defined as the diagram that shows a logical relation between sets.

The Venn diagram consists of circles to show the logical relation.

We have:

In a class of 70 pupils, 36 like tasty time, 34 like ice cream, 6 like both tasty time.

Total = 70 pupils

Number of like tasty time = 36

Number of like ice cream = 34

Number of like both = 6

Let x be the total number of pupils that like neither tasty-time nor ice cream

The number of pupils that like ice cream only =  34 - 6 = 28

The number of pupils that like tasty-time only = 36 - 6 = 30

From the Venn diagram:

28 + 30 + 6 + x = 70

x = 70 - 64

x = 6

Thus, the number of pupils that like neither tasty-time nor ice cream is 6 if in a class of 70 pupils, 36 like tasty time, 34 like ice cream, 6 like both tasty times.

Learn more about the Venn diagram here:

brainly.com/question/1024798

#SPJ2

2. About how much is 123.1 do you weigh in pounds? Estimate if you don't know☺ Find an online converter and find out how many kilograms that is.​

Answers

Answer:

123.1 pounds is vary long, and I don't want to repeat, so 55.8372207 repeat.

Step-by-step explanation:

If you have any questions regarding my answer, tell me them in the comments, and I will come answer them for you. Have a good day.

if 25 liters of milk can make 8 tin of cheese how many liters of milk would b needed to make 11 tin of cheese? ​

Answers

Answer:

Step-by-step explanation:

34.375 Liters of milk make 11 tins of cheese. Assuming that milk is available in whole liters only, 35 Liters of milk are needed.

Other Questions
Round 61,565 to thenearest hundred. paragraph writing about friend A sample of hydrogen gas H2 has a volume of 5.0 L and a pressure of 1.0 atm. What is the final pressure in atmospheres if the volume is decreased to 2 L with no change in temperature and amount of gas Which of the following is not true of the above examples of Neolithic ceramics?a.Patterns were engraved into each ceramic piece, and then white chalk was rubbed into the engravings so that the engravings stood out against the black background.b.These are examples of European pottery vessels, created between 3000-2000 BCE.c.These vessels were found in Spain, and are believed to have been created by the Spanish peoples around 3000 BCE.d.The technology to create ceramic vessels was understood long before any were actually created in Europe. 2. For each of the ionic compounds in the table below, name the compound and explain the rule that youused in formulating your name for the compound.Name:Rule for naming compound:-PbF4-NH4NO3-Li2S What are the coordinates of A after a 90 counterclockwise rotation about the origin. The most common alternate strategy for signing 16-19 is the:double digit method.touch and tell technique.2-digit strategytwist method. Which of the following statements are you most likely to hear during a well-conducted peer review?A Im sorry, I didnt get a chance to read Junes essaywhat is it about?B If you added more details, your essay would be more convincing.C Roberto never knows how to end his essays, as this essay shows.D Let me tell you about the essay I wroteits about football too! Explain how homologous and the fossil records support evolution. Thanks so much!!! Use the substitution method to solve the system of equations. Choose thecorrect ordered pair.y = 12x-8y = 8xA. (4, 12)B. (5, 11)C. (2,16)O D. (3, 15) Whats the answer pls 1. 2 How can Nepal be a developed country? ou own a portfolio that is 30 percent invested in Stock X, 20 percent in Stock Y, and 50 percent in Stock Z. The expected returns on these three stocks are 11 percent, 17 percent, and 13 percent, respectively. What is the expected return on the portfolio Your friend offers to place a bet with you. He will pay you $1 if your favorite sports team wins the game on Tuesday night. But you will pay him $3 if his team wins. Your team has an 80% chance of winning, whereas his only has a 20% chance. This bet is in your favor. True or False. The length of the base of a triangle is twice its height. If the area of the triangle is 441 square kilometers, find the height what is x(2)+y(6) if x=4 and y=(-4)Also, can anyone just talk? What is the area of the shaded part of the figure? What is the difference between heat capacity and specific heat capacity? I want the meaning please. True or False? Modern climate modeling on a global scale is dependent on accurate data and sophisticated algorithms designed to capture the complexity of weather and climate variables.Group of answer choicesTrueFalse The man is labeled a looter because of...A his physical appearance.Bhis clever break-in.his clothingDhis past crimes.